You are on page 1of 6
IITJAM | JEST | TIFR | CUET | NET | GATE Physics Prepkit Angular Momentum and Torque Problems Not hese are JEE Problems curated by PrepKit. Question 1 A particle of mass mis projected with a velocity vmaking an angle of 45° with the horizontal. ‘The magnitude of the angular momentum of the projectile about the point of projection when the particle is at its maximum height fis a) zero by mv" /(4/29) ©) mv'/o2a) a) mV2gh Solution (b) Lamon 2 vsin245° 2 2g 4g 2 > toa) 2 V2) (4g) 42g Question 2 A particle undergoes uniform circular motion. ‘About which point on the plane of the circle, the angular momentum of the particle remain conserved ? (@) Centre of circle (©) On the circumference of the circle (©) Inside the circle (@ Outside the circle Solution (a) In uniform circular motion, the only force acting on the particle is centripetal ( towards centre). Torque of this force about the centre is zero, Hence, angular momentum about the centre remains conserved. Question 3 ‘A mass m is moving with a constant velocity along.a line parallel to the x-axis, away from the origin. Its angular momentum with respect to the origin a) zero ») remains constant ©) goes on increasing d) goes on decreasing Here, n, = h= Solution (b) |= y~eonstant and |r|= (say) ‘Angular momentum ofthe particle about origin O will be aiven by L= rx p= m(r xy) ‘Therefore, angular momentum of particle about origin will remain constant, The direction oft x v also remains the same (negative). Question 4 An equilateral triangle ABC formed from a uniform wire has two small identical beads initially located at A. The triangle is set rotating about the vertical axis AO. Then the beads are released from rest simultaneously and allowed tosslide down, one along ABand other along AC as shown, Neglecting frictional effects, the quantities that are conserved as beads slides down are (@)angular velocity and total energy (kinetic and potential) (b) total angular momentum and total energy © angular velocity and moment of inertia about the axis of rotation (@) total angular momentum and moment of inertia about the axis of rotation Solution (6) Net external torque on the system is zero. ‘Therefore, angular momentum is conserved. Forces acting on the system are only conservative. Therefore, total mechanical energy Visit: www.myprepkit.in or Download the PrepKit App Page 1 Contact us: contact@myprepkit.in Whatsapp: 9971029358 IITJAM | JEST | TIFR | CUET | NET | GATE Physics PrepKit of the system is also conserved, Question 5 Aparticle moves in a circular path with decreasing speed. Choose the correct statement. (@) Angular momentum remains constant (b) Acceleration (a) is towards the centre (© Particle moves in a spiral path with decreasing radius (@) The direction of angular momentum remains constant Solution (d) L=mrxv) Direction of (F x v), hence the direction of angular momentum remains the same. Question 6 A bob of mass mattached to an inextensible string of length is suspended from a vertical support. The bob rotates in a horizontal circle with an angular speed w rad/s about the vertical support. About the point of suspension (@) angular momentum is conserved (b) angular momentum changes in magnitude but not in direction (©) angular momentum changes in direction but not in magnitude (@) angular momentum changes both in direction and magnitude Solution (¢) L=m(rxv) F and Dare continuously changing, So, Lis changing. Magnitude of Z will be same. Question 7 ‘The time dependence of the position ofa ati - 3}, particle of mass m=2 is given by Its angular momentum, with respect to the origin, at time ¢ = 2 is a) 36k b)- 34k) Q- 48% a) 40+) iven by So, angular momentum of particle of mass 2 ky a time 1281s L=(-2x6% 2") k= ~48i Question 8 A particle of mass 20 g is released with an initial velocity 5 m/s along the curve from the point A, as shown in the figure, The point A is at height from point B The particle slides along the frictionless surface. When the particle reaches point B its angular momentum about Owill be (Take, g= 10 m/s’) 9. ~~ CN n= 10m / Np 8 a) 8kg m/s b) 3kg m/s O2kg m/s 4) 6 kg m/s Solution (d) fae ile Jin}, + mgh = 4m 2 2 - V+ 2eb= or v= (9 42x 10 10= 295, = vy = 15ms Angular momentum about point ‘O", = 20. 107 x 15% 208 6kg-m? Question 9 A particle of mass mis projected at time t= 0 froma point Pon the ground with a speed v0 , at an angle of 45° to the horizontal. Find the ‘magnitude and direction of the angular Visit: www.myprepkit.in or Download the PrepKit App Page 2 Contact us: contact@myprepkit.in Whatsapp: 9971029358 IITJAM | JEST | TIFR | CUET | NET | GATE Physics Prepkit momentum of the particle about Pat time ©) L, remains constant while L,, varies with time tav/g. 4) L, and L, both vary with time Solution (my, */2/2g) Solution (¢) Interms of i, jand y Angular momentum ofa particle about a point i given by ae Lerepem(rxv) ane Bi Forty to Imi) = moi 9" corsa Dry mre i ey sermon, tohinnmguicn velsdecton For q "Now, angular momentum about point Pat given time él aan L=m(rxv) anf 8888 gem Ip [= (mvrsin 8) (m) (Reo) (sin 90° = (mo) Vag" 2¢~ 2 * w2e| 2g tude of Lp will remain constant but direction of Ly “Tus, magne of angular ad ‘ , magnitde of angular momentum isin ee ‘Wig u direction ic. in a direction perpendicular to paper nw Question 10 Asmall mass mis attached to a m: ‘ron a body about a given point is ‘equal to A x Z, where A is a constant whose other end is fixed at Pas ind Lis the angular momentum of the figure. The mass is undergoing itthat point. From this it follows that in the x- yplane with c (@ is perpendicular to Z atall instants of angular speed w. an (b) the component of 2 in the direction of A does not change with time (©) the magnitude of [ does not change with time (@ £ does not change with time Solution (a,c) eax i goth ae a From te propery fae prt cin i, 2p ot A ml. a) L, and L, do not vary with time ») L, varies with time while L, remains constant Visit: www.myprepkitin of Download the PrepKit App Page 3 Contact us: contact @myprepkctin Whatsapp: 9971029358 IITJAM | JEST | TIFR | CUET | NET | GATE Physics Prepkit (©) The given siution it very similar the relation Solution (a,c) av oe a Fe (ante [atr=Qy=0,r=9) ‘=> ais responsible for change in v. Ifa is always a=bpel=> Feds] perpendicular io thn direction of vehanges but its fea ‘magnitude remains constant. The two remains in one meds} plane Same is the case ere, (On negating, a mye ig [m= tks] ae 2 Fis always perpendicular to I. Therefore, magitud of Gsajyate=ts Lvl ean constant but ts deetion wil change, Let vase that Lea + yj avariale vecwor [r-0 ar-0) and A=ai+ bj+ ck = a constant vector Inthe aove eutions (i) Land ¥ always remain in one plane (here x=») (ipxand ye vas, bt (iF 9? coniant and Li-0 = F1L (po, end econ Now. is ays pedir A ence, Taso 7 oa — Siac nd yar coainonl chang onetines positive and sometimes negative). So, the above Condition can be fullfiled when a = = 0 or Aq ck ‘Therefore L,%and A are always mutually perpendicular. Hence, component of Lalong A is always constant at Question 12 Consider body of mass 1.0 kg at rest at the origin attime¢ = 0. Aforce P= (ati + Bj) is applied on the body, wherea = 1.0 N/s and = 1.0N. The torque acting on the body about the origin at time ¢ = 2,0s is t. Which of the following statements is (are) true? a) [t| = 1/3 1) The torque tis in the direction of the unit vector + k ©) The velocity of the body at ¢ = 1sis v =. (i+ 2imjs 4) The magnitude of displacement of the body at ¢ = Isis1/6m. Question 13 ‘The position vector r of particles of mass m is given by the following equation 7 = at'i+ pe j where @ = 10/3m/s", B = 5 m/s’ and m = 0.1kg. ‘Att = Is, which of the following statement(s) is/are true about the particle? a) the velocity vis given by ¥ = 101 + 10) m/s 'b) The angular momentum £ with respect to the origin is given by L =— 5/3 k Nms ©) The force Fis given by F = (i + 2/)N 4) The torque t with respect to the origin is given by t =— 20/3k Nm Solution (a,b,d) +80) See ieee ve asari +9 and a= = bad + fe 284 7 28) reat'i Atr=ls 0g 5 10} (0) v2 30D eiis2x 5xlj = Uoi+ 1o})mis @b=rx p= (Pxtts sx) xardoi +10) = (-38) Noms (OF =m = 04x (6x Dacisax = (i+ DN Visit: www.myprepkit.in or Download the PrepKit App Page 4 Contact us: contact@myprepkit.in Whatsapp: 9971029358 IITJAM | JEST | TIFR | CUET | NET | GATE Physics PrepKit (@t=rxF. (Fiss)

You might also like